Math, asked by jagadeeswarimk, 10 months ago

For which value of acute angle
Cos theta/1-sin theta+cos theta/1+sin theta=4 is true?

Answers

Answered by MaheswariS
23

\underline{\textbf{Given:}}

\mathsf{\dfrac{cos\,\theta}{1-sin\,\theta}+\dfrac{cos\,\theta}{1+sin\,\theta}=4}

\underline{\textbf{To find:}}

\textsf{The value of angle}\;\mathsf{\theta}\;\textsf{satisfying the given equation}

\underline{\textbf{Solution:}}

\underline{\textbf{Consider:}}

\mathsf{\dfrac{cos\,\theta}{1-sin\,\theta}+\dfrac{cos\,\theta}{1+sin\,\theta}=4}

\implies\mathsf{cos\,\theta\left(\dfrac{1}{1-sin\,\theta}+\dfrac{1}{1+sin\,\theta}\right)}=4}

\implies\mathsf{cos\,\theta\left(\dfrac{1+sin\,\theta+1+sin\,\theta}{(1-sin\,\theta)(1+sin\,\theta)}\right)=4}

\implies\mathsf{cos\,\theta\left(\dfrac{2}{1-sin^2\,\theta}\right)=4}

\implies\mathsf{cos\,\theta\left(\dfrac{2}{cos^2\,\theta}\right)=4}

\implies\mathsf{\dfrac{2}{cos\,\theta}=4}

\implies\mathsf{\dfrac{2}{4}=cos\,\theta}

\implies\mathsf{cos\,\theta=\dfrac{1}{2}}

\implies\boxed{\mathsf{\theta=60^\circ}}

Similar questions